how does addition of identity matrix to a square matrix changes determinant?












5












$begingroup$


Suppose there is $n times n$ matrix $A$. If we form matrix $B = A+I$ where $I$ is $n times n$ identity matrix, how does $|B|$ - determinant of $B$ - change compared to $|A|$? And what about the case where $B = A - I$?










share|cite|improve this question











$endgroup$

















    5












    $begingroup$


    Suppose there is $n times n$ matrix $A$. If we form matrix $B = A+I$ where $I$ is $n times n$ identity matrix, how does $|B|$ - determinant of $B$ - change compared to $|A|$? And what about the case where $B = A - I$?










    share|cite|improve this question











    $endgroup$















      5












      5








      5


      2



      $begingroup$


      Suppose there is $n times n$ matrix $A$. If we form matrix $B = A+I$ where $I$ is $n times n$ identity matrix, how does $|B|$ - determinant of $B$ - change compared to $|A|$? And what about the case where $B = A - I$?










      share|cite|improve this question











      $endgroup$




      Suppose there is $n times n$ matrix $A$. If we form matrix $B = A+I$ where $I$ is $n times n$ identity matrix, how does $|B|$ - determinant of $B$ - change compared to $|A|$? And what about the case where $B = A - I$?







      linear-algebra matrices determinant






      share|cite|improve this question















      share|cite|improve this question













      share|cite|improve this question




      share|cite|improve this question








      edited Mar 12 '18 at 8:20









      darij grinberg

      10.5k33062




      10.5k33062










      asked Dec 12 '12 at 9:08









      DDRDDR

      3112




      3112






















          6 Answers
          6






          active

          oldest

          votes


















          6












          $begingroup$

          In case you are interested, there is a result which expresses $det(A+B)$ in terms of $det(A)$ and $det (B)$, it is given by following $$det (A+B)=det(A)+det(B)+sum_{i=1}^{n-1}Gamma_n^idet(A/B^i)$$
          Where $Gamma_n^idet(A/B^i)$ is defined as a sum of the combination
          of determinants, in which the $i$ rows of $A$ are substituted
          by the corresponding rows of $B$. You can find the proof in this IEEE article: http://ieeexplore.ieee.org/stamp/stamp.jsp?tp=&arnumber=262036&userType=inst






          share|cite|improve this answer











          $endgroup$









          • 1




            $begingroup$
            Note: This formula is just a consequence of applying the multilinearity of the determinant once to each row (consecutively, so you end up with a sum of $2^n $ addends).
            $endgroup$
            – darij grinberg
            Mar 12 '18 at 8:20



















          3












          $begingroup$

          There is no simple answer. $P(lambda) = det(A-lambda I)$ is the characteristic polynomial of $A$. This is a polynomial of degree $n$: the coefficient of $lambda^n$ is $(-1)^n$ and the coefficient of $lambda^0$ is $det(A)$. The coefficients of other powers of $lambda$ are various functions of the entries of $A$. $det(A+I)$ and $det(A-I)$ are $P(-1)$ and $P(1)$; that's about all there is to say about them.






          share|cite|improve this answer









          $endgroup$





















            2












            $begingroup$

            As others already have pointed out, there is no simple relation. Here is one answer more for the intuition.
            Consider the (restricting) codition, that $A_{n times n}$ is diagonalizable, then $$det(A) = lambda_0 cdot lambda_1 cdot lambda_2 cdot cdots lambda _{n-1} $$
            Now consider you add the identity matrix. The determinant changes to
            $$det(B) = (lambda_0+1) cdot (lambda_1+1) cdot (lambda_2+1) cdot cdots (lambda _{n-1} +1)$$
            I think it is obvious how irregular the result depends on the given eigenvalues of A. If some $lambda_k=0$ then $det(A)=0$ but that zero-factor changes to $(lambda_k+1)$ and det(B) need not be zero. Other way round - if some factor $lambda_k=-1$ then the addition by I makes that factor $lambda_k+1=0$ and the determinant $det(B)$ becomes zero. If some $0 gt lambda_k gt -1$ then the determinant may change its sign...

            So there is no hope to make one single statement about the behave of B related to A - except that where @pritam linked to, or except you would accept a statement like $$det(A)=e_n(Lambda_n) to det(B)= sum_{j=0}^n e_j(Lambda) $$ where $ Lambda = {lambda_k}_{k=0..n-1} $ and $e_k(Lambda)$ denotes k'th elementary symmetric polynomial over $Lambda$... (And this is only for diagonalizable matrices)






            share|cite|improve this answer











            $endgroup$













            • $begingroup$
              You don't need diagonalizable in your arguments. You can always work with the Jordan form, or with the Schur decomposition, and reason exactly like you did.
              $endgroup$
              – Martin Argerami
              Dec 12 '12 at 13:07



















            1












            $begingroup$

            The characteristic polynomial $P_A(lambda)$ of a matrix $A$ is defined as
            $$
            P_A(lambda)=det(A-Ilambda)
            $$
            Therefore, $det(A)=P_A(0)$, while $det(A+I)=P_A(-1)$ and $det(A-I)=P_A(1)$.






            share|cite|improve this answer









            $endgroup$













            • $begingroup$
              Merry Christmas.
              $endgroup$
              – user1551
              Dec 12 '12 at 9:43










            • $begingroup$
              @user1551: and Happy New Year!
              $endgroup$
              – robjohn
              Dec 12 '12 at 10:02



















            0












            $begingroup$

            For I and A $Ntimes N$ matrices we have:



            $$det(I-A)=1-sum_{j}^{N}A_{jj}+sum_{ngeq 2}^{N}(-1)^{n}sum_{1leq j_{1}<...<j_{n}leq N}det((A_{j_{a},j_{b}})^{n}).$$






            share|cite|improve this answer









            $endgroup$





















              0












              $begingroup$

              I would like to contribute the following special case which I encountered when I was looking for an answer and stumbled upon this thread.



              If A=uv^intercal, then det(I+A) = 1+u^Tv






              share|cite|improve this answer









              $endgroup$













              • $begingroup$
                P.S Someone please help with formatting this to correct transposes. Didn't manage.
                $endgroup$
                – mexmex
                Dec 4 '18 at 9:54













              Your Answer





              StackExchange.ifUsing("editor", function () {
              return StackExchange.using("mathjaxEditing", function () {
              StackExchange.MarkdownEditor.creationCallbacks.add(function (editor, postfix) {
              StackExchange.mathjaxEditing.prepareWmdForMathJax(editor, postfix, [["$", "$"], ["\\(","\\)"]]);
              });
              });
              }, "mathjax-editing");

              StackExchange.ready(function() {
              var channelOptions = {
              tags: "".split(" "),
              id: "69"
              };
              initTagRenderer("".split(" "), "".split(" "), channelOptions);

              StackExchange.using("externalEditor", function() {
              // Have to fire editor after snippets, if snippets enabled
              if (StackExchange.settings.snippets.snippetsEnabled) {
              StackExchange.using("snippets", function() {
              createEditor();
              });
              }
              else {
              createEditor();
              }
              });

              function createEditor() {
              StackExchange.prepareEditor({
              heartbeatType: 'answer',
              autoActivateHeartbeat: false,
              convertImagesToLinks: true,
              noModals: true,
              showLowRepImageUploadWarning: true,
              reputationToPostImages: 10,
              bindNavPrevention: true,
              postfix: "",
              imageUploader: {
              brandingHtml: "Powered by u003ca class="icon-imgur-white" href="https://imgur.com/"u003eu003c/au003e",
              contentPolicyHtml: "User contributions licensed under u003ca href="https://creativecommons.org/licenses/by-sa/3.0/"u003ecc by-sa 3.0 with attribution requiredu003c/au003e u003ca href="https://stackoverflow.com/legal/content-policy"u003e(content policy)u003c/au003e",
              allowUrls: true
              },
              noCode: true, onDemand: true,
              discardSelector: ".discard-answer"
              ,immediatelyShowMarkdownHelp:true
              });


              }
              });














              draft saved

              draft discarded


















              StackExchange.ready(
              function () {
              StackExchange.openid.initPostLogin('.new-post-login', 'https%3a%2f%2fmath.stackexchange.com%2fquestions%2f256969%2fhow-does-addition-of-identity-matrix-to-a-square-matrix-changes-determinant%23new-answer', 'question_page');
              }
              );

              Post as a guest















              Required, but never shown

























              6 Answers
              6






              active

              oldest

              votes








              6 Answers
              6






              active

              oldest

              votes









              active

              oldest

              votes






              active

              oldest

              votes









              6












              $begingroup$

              In case you are interested, there is a result which expresses $det(A+B)$ in terms of $det(A)$ and $det (B)$, it is given by following $$det (A+B)=det(A)+det(B)+sum_{i=1}^{n-1}Gamma_n^idet(A/B^i)$$
              Where $Gamma_n^idet(A/B^i)$ is defined as a sum of the combination
              of determinants, in which the $i$ rows of $A$ are substituted
              by the corresponding rows of $B$. You can find the proof in this IEEE article: http://ieeexplore.ieee.org/stamp/stamp.jsp?tp=&arnumber=262036&userType=inst






              share|cite|improve this answer











              $endgroup$









              • 1




                $begingroup$
                Note: This formula is just a consequence of applying the multilinearity of the determinant once to each row (consecutively, so you end up with a sum of $2^n $ addends).
                $endgroup$
                – darij grinberg
                Mar 12 '18 at 8:20
















              6












              $begingroup$

              In case you are interested, there is a result which expresses $det(A+B)$ in terms of $det(A)$ and $det (B)$, it is given by following $$det (A+B)=det(A)+det(B)+sum_{i=1}^{n-1}Gamma_n^idet(A/B^i)$$
              Where $Gamma_n^idet(A/B^i)$ is defined as a sum of the combination
              of determinants, in which the $i$ rows of $A$ are substituted
              by the corresponding rows of $B$. You can find the proof in this IEEE article: http://ieeexplore.ieee.org/stamp/stamp.jsp?tp=&arnumber=262036&userType=inst






              share|cite|improve this answer











              $endgroup$









              • 1




                $begingroup$
                Note: This formula is just a consequence of applying the multilinearity of the determinant once to each row (consecutively, so you end up with a sum of $2^n $ addends).
                $endgroup$
                – darij grinberg
                Mar 12 '18 at 8:20














              6












              6








              6





              $begingroup$

              In case you are interested, there is a result which expresses $det(A+B)$ in terms of $det(A)$ and $det (B)$, it is given by following $$det (A+B)=det(A)+det(B)+sum_{i=1}^{n-1}Gamma_n^idet(A/B^i)$$
              Where $Gamma_n^idet(A/B^i)$ is defined as a sum of the combination
              of determinants, in which the $i$ rows of $A$ are substituted
              by the corresponding rows of $B$. You can find the proof in this IEEE article: http://ieeexplore.ieee.org/stamp/stamp.jsp?tp=&arnumber=262036&userType=inst






              share|cite|improve this answer











              $endgroup$



              In case you are interested, there is a result which expresses $det(A+B)$ in terms of $det(A)$ and $det (B)$, it is given by following $$det (A+B)=det(A)+det(B)+sum_{i=1}^{n-1}Gamma_n^idet(A/B^i)$$
              Where $Gamma_n^idet(A/B^i)$ is defined as a sum of the combination
              of determinants, in which the $i$ rows of $A$ are substituted
              by the corresponding rows of $B$. You can find the proof in this IEEE article: http://ieeexplore.ieee.org/stamp/stamp.jsp?tp=&arnumber=262036&userType=inst







              share|cite|improve this answer














              share|cite|improve this answer



              share|cite|improve this answer








              edited Dec 12 '12 at 11:11

























              answered Dec 12 '12 at 9:26









              pritampritam

              8,0671441




              8,0671441








              • 1




                $begingroup$
                Note: This formula is just a consequence of applying the multilinearity of the determinant once to each row (consecutively, so you end up with a sum of $2^n $ addends).
                $endgroup$
                – darij grinberg
                Mar 12 '18 at 8:20














              • 1




                $begingroup$
                Note: This formula is just a consequence of applying the multilinearity of the determinant once to each row (consecutively, so you end up with a sum of $2^n $ addends).
                $endgroup$
                – darij grinberg
                Mar 12 '18 at 8:20








              1




              1




              $begingroup$
              Note: This formula is just a consequence of applying the multilinearity of the determinant once to each row (consecutively, so you end up with a sum of $2^n $ addends).
              $endgroup$
              – darij grinberg
              Mar 12 '18 at 8:20




              $begingroup$
              Note: This formula is just a consequence of applying the multilinearity of the determinant once to each row (consecutively, so you end up with a sum of $2^n $ addends).
              $endgroup$
              – darij grinberg
              Mar 12 '18 at 8:20











              3












              $begingroup$

              There is no simple answer. $P(lambda) = det(A-lambda I)$ is the characteristic polynomial of $A$. This is a polynomial of degree $n$: the coefficient of $lambda^n$ is $(-1)^n$ and the coefficient of $lambda^0$ is $det(A)$. The coefficients of other powers of $lambda$ are various functions of the entries of $A$. $det(A+I)$ and $det(A-I)$ are $P(-1)$ and $P(1)$; that's about all there is to say about them.






              share|cite|improve this answer









              $endgroup$


















                3












                $begingroup$

                There is no simple answer. $P(lambda) = det(A-lambda I)$ is the characteristic polynomial of $A$. This is a polynomial of degree $n$: the coefficient of $lambda^n$ is $(-1)^n$ and the coefficient of $lambda^0$ is $det(A)$. The coefficients of other powers of $lambda$ are various functions of the entries of $A$. $det(A+I)$ and $det(A-I)$ are $P(-1)$ and $P(1)$; that's about all there is to say about them.






                share|cite|improve this answer









                $endgroup$
















                  3












                  3








                  3





                  $begingroup$

                  There is no simple answer. $P(lambda) = det(A-lambda I)$ is the characteristic polynomial of $A$. This is a polynomial of degree $n$: the coefficient of $lambda^n$ is $(-1)^n$ and the coefficient of $lambda^0$ is $det(A)$. The coefficients of other powers of $lambda$ are various functions of the entries of $A$. $det(A+I)$ and $det(A-I)$ are $P(-1)$ and $P(1)$; that's about all there is to say about them.






                  share|cite|improve this answer









                  $endgroup$



                  There is no simple answer. $P(lambda) = det(A-lambda I)$ is the characteristic polynomial of $A$. This is a polynomial of degree $n$: the coefficient of $lambda^n$ is $(-1)^n$ and the coefficient of $lambda^0$ is $det(A)$. The coefficients of other powers of $lambda$ are various functions of the entries of $A$. $det(A+I)$ and $det(A-I)$ are $P(-1)$ and $P(1)$; that's about all there is to say about them.







                  share|cite|improve this answer












                  share|cite|improve this answer



                  share|cite|improve this answer










                  answered Dec 12 '12 at 9:14









                  Robert IsraelRobert Israel

                  320k23210462




                  320k23210462























                      2












                      $begingroup$

                      As others already have pointed out, there is no simple relation. Here is one answer more for the intuition.
                      Consider the (restricting) codition, that $A_{n times n}$ is diagonalizable, then $$det(A) = lambda_0 cdot lambda_1 cdot lambda_2 cdot cdots lambda _{n-1} $$
                      Now consider you add the identity matrix. The determinant changes to
                      $$det(B) = (lambda_0+1) cdot (lambda_1+1) cdot (lambda_2+1) cdot cdots (lambda _{n-1} +1)$$
                      I think it is obvious how irregular the result depends on the given eigenvalues of A. If some $lambda_k=0$ then $det(A)=0$ but that zero-factor changes to $(lambda_k+1)$ and det(B) need not be zero. Other way round - if some factor $lambda_k=-1$ then the addition by I makes that factor $lambda_k+1=0$ and the determinant $det(B)$ becomes zero. If some $0 gt lambda_k gt -1$ then the determinant may change its sign...

                      So there is no hope to make one single statement about the behave of B related to A - except that where @pritam linked to, or except you would accept a statement like $$det(A)=e_n(Lambda_n) to det(B)= sum_{j=0}^n e_j(Lambda) $$ where $ Lambda = {lambda_k}_{k=0..n-1} $ and $e_k(Lambda)$ denotes k'th elementary symmetric polynomial over $Lambda$... (And this is only for diagonalizable matrices)






                      share|cite|improve this answer











                      $endgroup$













                      • $begingroup$
                        You don't need diagonalizable in your arguments. You can always work with the Jordan form, or with the Schur decomposition, and reason exactly like you did.
                        $endgroup$
                        – Martin Argerami
                        Dec 12 '12 at 13:07
















                      2












                      $begingroup$

                      As others already have pointed out, there is no simple relation. Here is one answer more for the intuition.
                      Consider the (restricting) codition, that $A_{n times n}$ is diagonalizable, then $$det(A) = lambda_0 cdot lambda_1 cdot lambda_2 cdot cdots lambda _{n-1} $$
                      Now consider you add the identity matrix. The determinant changes to
                      $$det(B) = (lambda_0+1) cdot (lambda_1+1) cdot (lambda_2+1) cdot cdots (lambda _{n-1} +1)$$
                      I think it is obvious how irregular the result depends on the given eigenvalues of A. If some $lambda_k=0$ then $det(A)=0$ but that zero-factor changes to $(lambda_k+1)$ and det(B) need not be zero. Other way round - if some factor $lambda_k=-1$ then the addition by I makes that factor $lambda_k+1=0$ and the determinant $det(B)$ becomes zero. If some $0 gt lambda_k gt -1$ then the determinant may change its sign...

                      So there is no hope to make one single statement about the behave of B related to A - except that where @pritam linked to, or except you would accept a statement like $$det(A)=e_n(Lambda_n) to det(B)= sum_{j=0}^n e_j(Lambda) $$ where $ Lambda = {lambda_k}_{k=0..n-1} $ and $e_k(Lambda)$ denotes k'th elementary symmetric polynomial over $Lambda$... (And this is only for diagonalizable matrices)






                      share|cite|improve this answer











                      $endgroup$













                      • $begingroup$
                        You don't need diagonalizable in your arguments. You can always work with the Jordan form, or with the Schur decomposition, and reason exactly like you did.
                        $endgroup$
                        – Martin Argerami
                        Dec 12 '12 at 13:07














                      2












                      2








                      2





                      $begingroup$

                      As others already have pointed out, there is no simple relation. Here is one answer more for the intuition.
                      Consider the (restricting) codition, that $A_{n times n}$ is diagonalizable, then $$det(A) = lambda_0 cdot lambda_1 cdot lambda_2 cdot cdots lambda _{n-1} $$
                      Now consider you add the identity matrix. The determinant changes to
                      $$det(B) = (lambda_0+1) cdot (lambda_1+1) cdot (lambda_2+1) cdot cdots (lambda _{n-1} +1)$$
                      I think it is obvious how irregular the result depends on the given eigenvalues of A. If some $lambda_k=0$ then $det(A)=0$ but that zero-factor changes to $(lambda_k+1)$ and det(B) need not be zero. Other way round - if some factor $lambda_k=-1$ then the addition by I makes that factor $lambda_k+1=0$ and the determinant $det(B)$ becomes zero. If some $0 gt lambda_k gt -1$ then the determinant may change its sign...

                      So there is no hope to make one single statement about the behave of B related to A - except that where @pritam linked to, or except you would accept a statement like $$det(A)=e_n(Lambda_n) to det(B)= sum_{j=0}^n e_j(Lambda) $$ where $ Lambda = {lambda_k}_{k=0..n-1} $ and $e_k(Lambda)$ denotes k'th elementary symmetric polynomial over $Lambda$... (And this is only for diagonalizable matrices)






                      share|cite|improve this answer











                      $endgroup$



                      As others already have pointed out, there is no simple relation. Here is one answer more for the intuition.
                      Consider the (restricting) codition, that $A_{n times n}$ is diagonalizable, then $$det(A) = lambda_0 cdot lambda_1 cdot lambda_2 cdot cdots lambda _{n-1} $$
                      Now consider you add the identity matrix. The determinant changes to
                      $$det(B) = (lambda_0+1) cdot (lambda_1+1) cdot (lambda_2+1) cdot cdots (lambda _{n-1} +1)$$
                      I think it is obvious how irregular the result depends on the given eigenvalues of A. If some $lambda_k=0$ then $det(A)=0$ but that zero-factor changes to $(lambda_k+1)$ and det(B) need not be zero. Other way round - if some factor $lambda_k=-1$ then the addition by I makes that factor $lambda_k+1=0$ and the determinant $det(B)$ becomes zero. If some $0 gt lambda_k gt -1$ then the determinant may change its sign...

                      So there is no hope to make one single statement about the behave of B related to A - except that where @pritam linked to, or except you would accept a statement like $$det(A)=e_n(Lambda_n) to det(B)= sum_{j=0}^n e_j(Lambda) $$ where $ Lambda = {lambda_k}_{k=0..n-1} $ and $e_k(Lambda)$ denotes k'th elementary symmetric polynomial over $Lambda$... (And this is only for diagonalizable matrices)







                      share|cite|improve this answer














                      share|cite|improve this answer



                      share|cite|improve this answer








                      edited Dec 12 '12 at 12:46

























                      answered Dec 12 '12 at 12:31









                      Gottfried HelmsGottfried Helms

                      23.3k24498




                      23.3k24498












                      • $begingroup$
                        You don't need diagonalizable in your arguments. You can always work with the Jordan form, or with the Schur decomposition, and reason exactly like you did.
                        $endgroup$
                        – Martin Argerami
                        Dec 12 '12 at 13:07


















                      • $begingroup$
                        You don't need diagonalizable in your arguments. You can always work with the Jordan form, or with the Schur decomposition, and reason exactly like you did.
                        $endgroup$
                        – Martin Argerami
                        Dec 12 '12 at 13:07
















                      $begingroup$
                      You don't need diagonalizable in your arguments. You can always work with the Jordan form, or with the Schur decomposition, and reason exactly like you did.
                      $endgroup$
                      – Martin Argerami
                      Dec 12 '12 at 13:07




                      $begingroup$
                      You don't need diagonalizable in your arguments. You can always work with the Jordan form, or with the Schur decomposition, and reason exactly like you did.
                      $endgroup$
                      – Martin Argerami
                      Dec 12 '12 at 13:07











                      1












                      $begingroup$

                      The characteristic polynomial $P_A(lambda)$ of a matrix $A$ is defined as
                      $$
                      P_A(lambda)=det(A-Ilambda)
                      $$
                      Therefore, $det(A)=P_A(0)$, while $det(A+I)=P_A(-1)$ and $det(A-I)=P_A(1)$.






                      share|cite|improve this answer









                      $endgroup$













                      • $begingroup$
                        Merry Christmas.
                        $endgroup$
                        – user1551
                        Dec 12 '12 at 9:43










                      • $begingroup$
                        @user1551: and Happy New Year!
                        $endgroup$
                        – robjohn
                        Dec 12 '12 at 10:02
















                      1












                      $begingroup$

                      The characteristic polynomial $P_A(lambda)$ of a matrix $A$ is defined as
                      $$
                      P_A(lambda)=det(A-Ilambda)
                      $$
                      Therefore, $det(A)=P_A(0)$, while $det(A+I)=P_A(-1)$ and $det(A-I)=P_A(1)$.






                      share|cite|improve this answer









                      $endgroup$













                      • $begingroup$
                        Merry Christmas.
                        $endgroup$
                        – user1551
                        Dec 12 '12 at 9:43










                      • $begingroup$
                        @user1551: and Happy New Year!
                        $endgroup$
                        – robjohn
                        Dec 12 '12 at 10:02














                      1












                      1








                      1





                      $begingroup$

                      The characteristic polynomial $P_A(lambda)$ of a matrix $A$ is defined as
                      $$
                      P_A(lambda)=det(A-Ilambda)
                      $$
                      Therefore, $det(A)=P_A(0)$, while $det(A+I)=P_A(-1)$ and $det(A-I)=P_A(1)$.






                      share|cite|improve this answer









                      $endgroup$



                      The characteristic polynomial $P_A(lambda)$ of a matrix $A$ is defined as
                      $$
                      P_A(lambda)=det(A-Ilambda)
                      $$
                      Therefore, $det(A)=P_A(0)$, while $det(A+I)=P_A(-1)$ and $det(A-I)=P_A(1)$.







                      share|cite|improve this answer












                      share|cite|improve this answer



                      share|cite|improve this answer










                      answered Dec 12 '12 at 9:15









                      robjohnrobjohn

                      266k27305626




                      266k27305626












                      • $begingroup$
                        Merry Christmas.
                        $endgroup$
                        – user1551
                        Dec 12 '12 at 9:43










                      • $begingroup$
                        @user1551: and Happy New Year!
                        $endgroup$
                        – robjohn
                        Dec 12 '12 at 10:02


















                      • $begingroup$
                        Merry Christmas.
                        $endgroup$
                        – user1551
                        Dec 12 '12 at 9:43










                      • $begingroup$
                        @user1551: and Happy New Year!
                        $endgroup$
                        – robjohn
                        Dec 12 '12 at 10:02
















                      $begingroup$
                      Merry Christmas.
                      $endgroup$
                      – user1551
                      Dec 12 '12 at 9:43




                      $begingroup$
                      Merry Christmas.
                      $endgroup$
                      – user1551
                      Dec 12 '12 at 9:43












                      $begingroup$
                      @user1551: and Happy New Year!
                      $endgroup$
                      – robjohn
                      Dec 12 '12 at 10:02




                      $begingroup$
                      @user1551: and Happy New Year!
                      $endgroup$
                      – robjohn
                      Dec 12 '12 at 10:02











                      0












                      $begingroup$

                      For I and A $Ntimes N$ matrices we have:



                      $$det(I-A)=1-sum_{j}^{N}A_{jj}+sum_{ngeq 2}^{N}(-1)^{n}sum_{1leq j_{1}<...<j_{n}leq N}det((A_{j_{a},j_{b}})^{n}).$$






                      share|cite|improve this answer









                      $endgroup$


















                        0












                        $begingroup$

                        For I and A $Ntimes N$ matrices we have:



                        $$det(I-A)=1-sum_{j}^{N}A_{jj}+sum_{ngeq 2}^{N}(-1)^{n}sum_{1leq j_{1}<...<j_{n}leq N}det((A_{j_{a},j_{b}})^{n}).$$






                        share|cite|improve this answer









                        $endgroup$
















                          0












                          0








                          0





                          $begingroup$

                          For I and A $Ntimes N$ matrices we have:



                          $$det(I-A)=1-sum_{j}^{N}A_{jj}+sum_{ngeq 2}^{N}(-1)^{n}sum_{1leq j_{1}<...<j_{n}leq N}det((A_{j_{a},j_{b}})^{n}).$$






                          share|cite|improve this answer









                          $endgroup$



                          For I and A $Ntimes N$ matrices we have:



                          $$det(I-A)=1-sum_{j}^{N}A_{jj}+sum_{ngeq 2}^{N}(-1)^{n}sum_{1leq j_{1}<...<j_{n}leq N}det((A_{j_{a},j_{b}})^{n}).$$







                          share|cite|improve this answer












                          share|cite|improve this answer



                          share|cite|improve this answer










                          answered Mar 12 '18 at 0:47









                          Thomas KojarThomas Kojar

                          1155




                          1155























                              0












                              $begingroup$

                              I would like to contribute the following special case which I encountered when I was looking for an answer and stumbled upon this thread.



                              If A=uv^intercal, then det(I+A) = 1+u^Tv






                              share|cite|improve this answer









                              $endgroup$













                              • $begingroup$
                                P.S Someone please help with formatting this to correct transposes. Didn't manage.
                                $endgroup$
                                – mexmex
                                Dec 4 '18 at 9:54


















                              0












                              $begingroup$

                              I would like to contribute the following special case which I encountered when I was looking for an answer and stumbled upon this thread.



                              If A=uv^intercal, then det(I+A) = 1+u^Tv






                              share|cite|improve this answer









                              $endgroup$













                              • $begingroup$
                                P.S Someone please help with formatting this to correct transposes. Didn't manage.
                                $endgroup$
                                – mexmex
                                Dec 4 '18 at 9:54
















                              0












                              0








                              0





                              $begingroup$

                              I would like to contribute the following special case which I encountered when I was looking for an answer and stumbled upon this thread.



                              If A=uv^intercal, then det(I+A) = 1+u^Tv






                              share|cite|improve this answer









                              $endgroup$



                              I would like to contribute the following special case which I encountered when I was looking for an answer and stumbled upon this thread.



                              If A=uv^intercal, then det(I+A) = 1+u^Tv







                              share|cite|improve this answer












                              share|cite|improve this answer



                              share|cite|improve this answer










                              answered Dec 4 '18 at 9:54









                              mexmexmexmex

                              1236




                              1236












                              • $begingroup$
                                P.S Someone please help with formatting this to correct transposes. Didn't manage.
                                $endgroup$
                                – mexmex
                                Dec 4 '18 at 9:54




















                              • $begingroup$
                                P.S Someone please help with formatting this to correct transposes. Didn't manage.
                                $endgroup$
                                – mexmex
                                Dec 4 '18 at 9:54


















                              $begingroup$
                              P.S Someone please help with formatting this to correct transposes. Didn't manage.
                              $endgroup$
                              – mexmex
                              Dec 4 '18 at 9:54






                              $begingroup$
                              P.S Someone please help with formatting this to correct transposes. Didn't manage.
                              $endgroup$
                              – mexmex
                              Dec 4 '18 at 9:54




















                              draft saved

                              draft discarded




















































                              Thanks for contributing an answer to Mathematics Stack Exchange!


                              • Please be sure to answer the question. Provide details and share your research!

                              But avoid



                              • Asking for help, clarification, or responding to other answers.

                              • Making statements based on opinion; back them up with references or personal experience.


                              Use MathJax to format equations. MathJax reference.


                              To learn more, see our tips on writing great answers.




                              draft saved


                              draft discarded














                              StackExchange.ready(
                              function () {
                              StackExchange.openid.initPostLogin('.new-post-login', 'https%3a%2f%2fmath.stackexchange.com%2fquestions%2f256969%2fhow-does-addition-of-identity-matrix-to-a-square-matrix-changes-determinant%23new-answer', 'question_page');
                              }
                              );

                              Post as a guest















                              Required, but never shown





















































                              Required, but never shown














                              Required, but never shown












                              Required, but never shown







                              Required, but never shown

































                              Required, but never shown














                              Required, but never shown












                              Required, but never shown







                              Required, but never shown







                              Popular posts from this blog

                              Quarter-circle Tiles

                              build a pushdown automaton that recognizes the reverse language of a given pushdown automaton?

                              Mont Emei